Let X_1, \ldots , X_ n \stackrel{iid}{\sim } \mathbf{P} for some distribution \, \mathbf{P}\,, and let F denote its cdf. Let F_ n denote the empirical cdf. Then it holds for every t \in \mathbb {R} that

\sqrt{n} (F_ n(t) - F(t)) \xrightarrow [n \to \infty ]{(d)} \mathcal{N}(0, \sigma ^2)

for any fixed t and for some asymptotic variance \sigma ^2.

What theorem implies the above convergence statement?

Central limit theorem.

Law of large numbers.

Glivenko-Cantelli theorem.
unanswered
Which of the following is \sigma ^2? Note that \sigma ^2 is dependent on t.

F(t)

1 - F(t)

\sqrt{F(t)(1 - F(t))}

F(t)(1 - F(t))
unanswered
What is the asymptotic variance \sigma ^2 of F_ n(0) in terms of the values of the cdf F? (Enter F(x) for F(x) for any numerical value x.)

\, \sigma ^2 = \,

F(0)\cdot(1-F(0)).

\sigma ^2 = F(t)(1 - F(t))